Sei sulla pagina 1di 76

JEE Knockout Test Solution

1. Time duration for this test is 180 minutes. This test consists of 90 questions. The
maximum marks are 360.
2. There are three parts in the question paper, namely, Part A: Mathematics, Part B:
Physics and Part C: Chemistry. Each question is awarded 4 (four) marks for correct
response.
3. One-fourth (1/4) marks will be deducted for indicating incorrect response of each
question. No deduction from the total score will be made if no response is indicated for
an item in the answer sheet.
4. This test contains 90 Multiple Choice Questions with single correct answer. Each
Question has four choices (1), (2), (3) & (4), out of which Only One is Correct.
5. Filling up more than one response in any question will be treated as wrong response and
marks for wrong response will be deducted accordingly as per the instruction 3 above.

Part A Mathematics

Question 1: If
14
7
, then find .
3 4
i
Z Z
i
+
=
+

(1) 2
7
(2) (2)
7

(3) (2
7
)

i (4) (2
7
) i
Chapter: Complex Numbers
Level of Difficulty: Easy
Solution: We have
7
3 4
i
Z
i
+
=
+

Simplifying (i.e., rationalizing the denominator), we get
7 3 4 21 4 28 3
3 4 3 4 9 16
25 25
25
i i i i
i i
i
+ + +
=
+ +

=

= 1 i
Therefore,

14
14
2 7
2 7
7
7
(1 )
3 4
[(1 ) ]
(1 2 )
( 2 )
i
i
i
i
i i
i
+ | |
=
|
+
\ .
=
= +
= +

Correct Option: (3).

Question 2: If 4 4 10, Z Z + + = then the difference between the maximum and the minimum values of
Z is
(1) 2 (2) 3
(3) 41 5 (4) 0
Chapter: Complex Numbers
Level of Difficulty: Hard
Solution
4 4 10 Z Z + + =
Ps + Ps' = 2a
which implies that foci at 4 and 4 and a = 5 as shown in the following figure.

Now,
b
2
= 25(1 e
2
)
= 25 (5e)
2

= 25 16 = 9
b = 3
Z lies on the ellipse circumference Z denotes the distance from the origin. Therefore,
bi
-bi

max
min
5
3
Z
Z
=
=

Thus, the difference between the maximum and the minimum values of Z is
max min
5 3 2 Z Z = =
Correct Option: (1).

Question 3: If [x]
2
7[x] + 10 < 0 and 4[y]
2
16[y] + 7 < 0, then [x + y] cannot be ([ ] denotes greatest
integer function)
(1) 7 (2) 8
(3) 9 (4) Both (2) and (3)
Chapter: Quadratic Equations and Expressions
Level of Difficulty: Moderate
Solution: We have
2
[ ] 7[ ] 10 0 x x + <
([ ] 5) ([ ] 2) 0 x x <
2 [ ] 5 x < <
[ ] 3or 4 x =
[3, 5) x e
and
2
4[ ] 16[ ] 7 0 y y + <
(2[ ] 7)(2[ ] 1) 0 y y <
1 7
[ ]
2 2
y < <
[ ] 1 or 2 or 3 y =
[1, 4) y e
Therefore,
[4, 9)
[ ] {4, 5, 6, 7,8}
x y
x y
+ e
+ e

Hence, [x + y] cannot be 9.
Correct Option: (3).

Question 4: If a, b and c are the roots of the equation x
3
+ 2x
2
+ 1= 0, find .
a b c
b c a
c a b

(1) 8 (2) 8
(3) 0 (4) 2
Chapter: Matrices, Determinants and System of Equations
Level of Difficulty: Moderate
Solution: As a, b and c are the roots of x
3
+ 2x
2
+ 1 = 0, we have
a + b + c = 2
ab + bc + ca = 0
abc = 1
Now, for finding the value of
a b c
b c a
c a b
, evaluating using first row, we get
a(bc a
2
) b(b
2
ac) + c(ab c
2
) = abc a
3
b
3
+ abc + abc c
3

= 3abc a
3
b
3
c
3

= (a
3
+ b
3
+ c
3
3abc)
= (a + b + c) (a
2
+ b
2
+ c
2
ab bc ca)
= (2) [(2)
2
3(0)] = 8
Correct Option: (1).

Question 5: If
1 2 cos sin
, ,
0 1 sin cos
T
A P Q P AP
u u
u u
| | | |
= = =
| |

\ . \ .
, find
2014
.
T
PQ P
(1)
2014
1 2
0 1
| |
|
|
\ .
(2)
1 4028
0 1
| |
|
\ .

(3)
2013 2014 2013
( )
T
P A P (4)
2014 T
P A P
Chapter: Matrices, Determinants and System of Equations
Level of Difficulty: Hard
Solution: If
1 2 cos sin
, , ,
0 1 sin cos
T
A P Q P AP
u u
u u
| | | |
= = =
| |

\ . \ .
we have
PQ
20/4
P
T
( )( ) ( )
2014 times
T T T T
P P AP P AP P AP P
=
( ) ( ) ( ) ( ) ( )
T T T T T
PP A PP A PP PP A PP =

Matrix multiplication is associative:
cos sin cos sin
sin cos sin cos
T
PP
u u u u
u u u u
| | | |
=
| |
+ \ . \ .


2
1 0
0 1
I
| |
= =
|
\ .

Hence, PQ
2014
P
T
= A
2014

2
1 2 1 2 1 2 1 4
0 1 0 1 0 1 0 1
A A
| | | | | | | |
= = =
| | | |
\ . \ . \ . \ .

3
1 4 1 2 1 6
0 1 0 1 0 1
A
| | | | | |
= =
| | |
\ . \ . \ .

4
1 6 1 2 1 8
0 1 0 1 0 1
A
| | | | | |
= =
| | |
\ . \ . \ .

2014
1 2 1 4028
and =
0 1 0 1
n
n
A A
| | | |
=
| |
\ . \ .

Correct Option: (2).

Question 6: Find the value of
3 3 3 3 3 3 3 3 3 3
2 6 12 20
1 1 2 1 2 3 1 2 3 4
+ + + +
+ + + + + +
up to 60 terms.
(1) 2 (2)
1
2

(3) 4 (4)
1
4

Chapter: Progressions, Sequences and Series
Level of Difficulty: Moderate
Solution
3 3 3 3 3 3 3 3 3 3
2 6 12 20
...
1 1 2 1 2 3 1 2 3 4
+ + + +
+ + + + + +

3 3 3 3 3 3
1 2 2 3 3 4
...
1 1 2 1 2 3

= + + +
+ + +

3 3 3
1
( 1)
lim
1 2 ...
n
n
n n
n

+
=
+ + +


2
1
( 1)
lim
( 1)
2
n
n
n n
n n

+
=
+ | |
|
\ .



1
1
lim 4
( 1)
n
n n n
=
+


1
1 1
4 lim
1
n
n n n
| |
=
|
\ .
+


1 1 1 1 1 1
4 lim ...
1 2 2 3 1 n n n
| |
= + + +
|
\ .
+

4 lim 4
1 n
n
n
= =
+

Correct Option: (3).

Question 7: In the expansion of (1 + x)
2
(1 + y)
3
(1 + z)
4
(1 + w)
5
, the sum of the coefficient of the terms
of degree 12 is
(1) 61 (2) 71
(3) 81 (4) 91
Chapter: Binomial Theorem
Level of Difficulty: Hard
Solution
(1 + x)
2
(1 + y)
3
(1 + z)
4
(1 + w)
5

General term
2 3 4 5 a b d e
a b d e
C C C C x
+ + +
=
2 3 4 5 14 14
12 12
12
14 13
or
2
a b d e
a b d e
C C C C C C
+ + + =

= = = 91
Correct Option: (4).

Question 8: Let S be the set of all functions from the set {1, 2, , 10} to itself. One function is selected
from S, the probability that the selected function is oneone onto is
(1)
9
9!
10
(2)
1
10

(3)
100
10!
(4)
10
9!
10

Chapter: Probability
Level of Difficulty: Moderate
Solution: Let f be function from {1, 2,,10} to itself total functions possible is 10
10
. The number of
oneone onto functions possible is 10!
Hence, the probability of selected function to be oneone onto is

10 9
10! 9!
10 10
=
Correct Option: (1).

Question 9: Two friends visit a restaurant randomly during 5 pm to 6 pm. Among the two, whoever
comes first waits for 15 min and then leaves. The probability that they meet is
(1)
1
4
(2)
1
16

(3)
7
16
(4)
9
16

Chapter: Probability
Level of Difficulty: Hard
Solution: Let the friends come to the restaurant at 5 h x min and 5 h y min, respectively, where
, [0, 60] x ye .
Hence, the sample space consists of all points (x, y) lying in 60 60 square as shown above and for
favourable cases, 15 x y s , that is, 15 x y 15 which is shown by shaded region in the graph
shown below.

Hence, the probability that they will meet is given by
1
45 45
2
1
60 60
o

2
3 7
1
4 16
| |
= =
|
\ .

Correct Option: (3).


Question 10: If ( ) ( ) ( ) r a m n b n l c l m = + + and [ ] 4 l mn = , find .
( )
a b c
r l m n
+ +
+ +

(1)
1
4
(2)
1
2

(3) 1 (4) 2
Chapter: Vectors
Level of Difficulty: Moderate
Solution
( ) ( ) ( ) r a m n b n l c l m = + +
where
[ ] 4 l mn =
4 r l a =
4 r m b =
4 r n c =
which imply that
1
4 ( )
a b c
r l m n
+ +
=
+ +

Correct Option
The correct option is (1).

Question 11: The volume of tetrahedron, for which three co-terminus edges are , and a b c , is k units.
Then, the volume of a parallelepiped formed by , 2 a b b c + and 3a c is
(1) 6k (2) 7k
(3) 30k (4) 42k
Chapter: Vectors
Level of Difficulty: Hard
Solution
The volume tetrahedron is given by
1
[ ] [ ] 6
6
k ab c ab c k = =
The volume of parallelepiped is given by
[ 2 3 ] a b b c a c +
[ 2 3 ] [ 2 3 ] a b c ac b b c a c = + + +

[ 3 ] [ 2 3 ] [ 3 ] [ 2 3 ] a b a c a c a c bb a c b c a c = + + +
[ ] [ 2 3 ] a b c b c a = +
[ ] 6[ ] a b c a b c =
7[ ] a b c =
Volume is 42k.
Correct Option: (4).

Question 12: Statement 1: Negation of the statement if price increases, then demand falls is price
increases and demand does not fall
Statement 2: Negative of p q is ~ p q v .
(1) Statement 1 is True; Statement 2 is True; Statement 2 is a correct explanation for Statement 1.
(2) Statement 1 is True; Statement 2 is true; Statement 2 is not a correct explanation for Statement 1.
(3) Statement 1 is True; Statement 2 is False.
(4) Statement 1 is False; Statement 2 is True.
Chapter: Mathematical Reasoning
Level of Difficulty: Easy
Solution: The negation of p q is ~ . p q . Hence, the negation of if price increases then demand falls
is price increases and demand does not falls.
Statement 1 is correct, but Statement 2 is incorrect.
Correct Option: (3).

Question 13: Simultaneously two dice are thrown five times. If getting a sum as multiple of 3 is
considered as success, then the variance of the distribution of number of successes is
(1)
5
3
(2)
10
3

(3)
10
9
(4)
140
121

Chapter: Statistics
Level of Difficulty: Hard
Solution
The sum as multiple of 3, possibilities are 3, 6, 9 and 12, that is, (1, 2), (2, 1), (1, 5) (2, 4) (3, 3) (4, 2) (5,
1) (3, 6) (4, 5) (5, 4) (6, 3) and (6, 6). There are 12 possibilities and hence the probability of success is

12 1
36 3
=
It is a binomial distribution with n = 5 and
1
3
p = , which implies that the variance is
1 2 10
5
3 3 9
npq = =
Correct Option: (3).

Question 14: If 270 < < 360, then find 2 2(1 cos ) . u + +
(1) 2sin
4
u | |

|
\ .
(2) 2sin
4
u | |
|
\ .

(3) 2sin
4
u
(4) 2cos
4
u

Chapter: Trigonometric Ratios and Identities
Level of Difficulty: Moderate
Solution: When, 270 < < 360, we have
2(1 cos ) u +
2
2 2cos
2
u | |
=
|
\ .

which is non-negative. Now, the above equation can be written as
2(1 cos ) u + 2 cos
2
u
=
2cos
2
u
= cos 0when135 180
2 2
u u | |
< < <
|
\ .

Now, let us consider that
2 2(1 cos ) u + +
which is non-negative. That is,
2
2 2(1 cos ) 2 2cos
2
2 1 cos
2
2 2sin
4
2 sin
4
u
u
u
u
u
+ + =
=
=
=



135
2sin sin 0when 90
4 4 2 4
u u u | |
= > < <
|
\ .

Correct Option: (2).

Question 15: The number of solutions of sin
1
x + sin
1
(1 + x) = cos
1
x is/are
(1) 0 (2) 1
(3) 2 (4) infinite
Chapter: Inverse Trigonometric Functions
Level of Difficulty: Easy
Solution
sin
1
(1 + x) is defined for x < 0 and
1 1
sin cos 1 1.
2
x x x
t

= s s
The given equation is
1 1 1
sin sin (1 ) cos x x x

+ + =
which can be written as
1 1 1
cos cos (1 ) cos
2 2
x x x
t t

+ + =
1 1
cos (1 ) 2cos x x t

+ =
1 1 2
cos ( 1 ) 2 cos (2 1) x x t

=
1 1 2
cos ( 1 ) cos (2 1) 2 x x t

+ =
1 1 2
cos ( 1 ) cos (2 1) x x t

= =
2
1 2 1 1 x x = =
0 x =
Which implies that the total number of solutions sin
1
x + sin
1
(1 + x) = cos
1
x is only one.
Correct Option: (2).

Question 16: If P(A) is the power set of set A, which of the following statements is false?
(1) { : } A B x x A x B _ e e (2) A B B A _ _
(3) N I Q R C c c c c (4) ( ) { : } P A X X A = c
Chapter: Sets
Level of Difficulty: Easy
Solution

1. : x x A x B e e means that all elements exist in set A also exist in set B. Similarly, A B _ means
that all elements exist in set A also exist in set B. The Venn diagram can be drawn as shown in
any of the following two figures:


Therefore, option (1) is correct.
2. A B _ means A is a subset of B or B A _ means that B is a super set of A. Therefore, option (2) is
correct.
3. It is a well-known fact that N I c or N I _ and hence both are correct. Therefore, option (3) is
correct.
4. The statement ( ) { : } P A X X A = c is wrong and the correct statement is ( ) { : } P A X X A = _ , that
is, set X = set A is also included. Therefore, option (4) is incorrect or false.
Correct Option: (4).

Question 17: The value of
2
lim ( , , , , {0})
x
ax bx c
a b c d e
dx e
+ +
e
+
depends on the sign of
(1) a only (2) d only
(3) a and d only (4) a, b and d only
Chapter: Limits
Level of Difficulty: Moderate
Solution
2
( / )
lim lim
( / )
lim
x x
x
ax bx c ax b c x
dx e d e x
a b
x
d d

+ + + +
=
+ +
| |
= +
|
\ .


= + if
a
d
| |
|
\ .
is positive.
= if
a
d
| |
|
\ .
is negative.

Correct Option: (3).
Alternate Solution
2 2
2
( / ) ( / )
lim lim
( / ) ( / )
x x
ax bx c a b x c x
dx e d x e x

+ + + +
=
+ +

Here,
2
e d
x x
<< . Therefore,
2
lim lim
/
if 0 and 0
if 0
if 0 and 0
0
if 0 and 0
if 0
0 if 0 and 0
x x
ax bx c a
dx e d x
a d
a
d
a d
a a d
d
a d

+

+ +
=
+
+ > >

>

< >

=

> <

<

+ < <



Question 18: If y = (x 3)(x 2)(x 1)x(x + 1)(x + 2)(x + 3), then
2
2
d y
dx
at x = 1 is
(1) 101 (2) 48
(3) 56 (4) 190
Chapter: Differentiation
Level of Difficulty: Easy
Solution
We have
2 2 2
( 9)( 4)( 1) y x x x x =

6 4 2
( 14 (49) 36) x x x x = +

7 5 3
14 49 36 x x x x = +
Therefore,
6 4 2
7 70 147 36
dy
x x x
dx
= +
Thus,
2
5 3
2
42 280 294
d y
x x x
dx
= +
2
2
1
42 280 294
x
d y
dx
=
= + = 56
Correct Option: (3).


Question 19: If
sin cos( )
2
dy x
x
dx
t
t
| |
=
|
\ .
, then y is strictly increasing in
(1) (3, 4) (2)
5 7
,
2 2
| |
|
\ .

(3) (2, 3) (4)
1 3
,
2 2
| |
|
\ .

Chapter: Applications of Derivatives
Level of Difficulty: Moderate
Solution: Let us draw the graph of
( ) sin
2
f x x
t | |
=
|
\ .

and
( ) cos( ) g x xt =

on the same xy-plane as shown in the following figure.




From this graphical representation, it is clear that y is strictly increasing in

5 7
,
2 2
| |
|
\ .

f(x)
x
y

Because for all values of x,
5 7
2 2
x < <
That is,
sin 0
2
x
t | |
<
|
\ .

and
cos ( ) 0 xt <
which imply that
0
dy
dx
>
which means that y is strictly increasing.

Correct Option: (2).

Question 20: Given that
( )
( )
( )
( )
( )
2 1 1
2 2
2 3
2 1 (1 )
2 1 1 1
n n n
n
dx x dx
n x
n x x

= +
+
+ +
} }
. Find the value of
( )
1
4
0
2
.
1
dx
x +
}

(You may or may not use reduction formula given.)
(1)
11 5
48 64
t
+ (2)
11 5
48 32
t
+
(3)
1 5
24 64
t
+ (4)
1 5
96 32
t
+
Chapter: Indefinite Integration
Level of Difficulty: Moderate
Solution

( )
3
1
1 1
4 2 4 1
0 0 2)
2
0
1
1
3 2 2 2 2
0
0
1
1
2 2
0
0
1
5
6 2(4 1)(1 )
(1
1
1 5 5 3
0 .
6 6 4 6(2) 2(2)(1 ) (1 )
1 5 1 5 5 1
0
48 6 16 8 8 2 2(1)(1 ) 1
1 5 5 1 5
0 tan
48 6 16 8 4 16
dx x dx
x
x
x
x dx
x x
x dx
x x
x

(
= +
(
+
+
+
| | (
= + +
| (
+ +
\ .
(
| | (
= + + +
( |
(
+ + \ .

| |
= + + +
|

\ .
} }
}
}
1
0
7 5 5
0
6 16 8 4 16 4
22 5
6 16 64
11 5
48 64
t
t
t
(

(
= + +
(


= +

= +

Correct Option: (1).
Alternate Solution
( )
1
4
0
2
1
dx
I
x
=
+
}

Put tan x u = ; therefore,
2
sec dx d u u = .
( )
2
4
8
0
sec
sec
d
t
u u
I =
u
}

That is,

( )
/ 4
6
0
2
/ 4
0
/ 4 / 4 / 4
2 2
0 0 0
/ 4 / 4 / 4
0 0 0
/ 4 / 4
0 0
(cos )
3cos cos3
4
9 1 3
cos (cos3 ) cos cos3
16 16 8
4 cos 2
9 1 cos 2 1 1 cos6 3
16 2 16 2 8 2
9 sin 2 1 sin6
32 2 16 2 6
d
d
d d
cos
d d
t
t
t t t
t t t
t t
I = u u
u u
u
u u u u u u
u u
u u
u u
u u
u u
+ | |
=
|
\ .
= + +
+
+ +
= + +
( (
= + + + +
( (


}
}
} } }
} } }
/ 4
0
3 sin 4 sin 2
8 2 4 2
9 1 1 1 3 1
0 0
32 4 2 16 2 4 6 8 2 2
5 11
64 48
t
u u
t t
t
(
+
(


| | ( ( (
= + + + +
|
( ( (

\ .
= +



Question 21: If the length of the normal for each point on a curve is equal to the radius vector, then the
curve
(1) is a circle passing through origin
(2) is a circle having centre at origin and radius > 0
(3) is a circle having centre on x-axis and touching y-axis
(4) is a circle having centre on y-axis and touching x-axis
Chapter: Differential Equations
Level of Difficulty: Hard
Solution: The length of the normal is
2
1
dy
y
dx
| |
+
|
\ .

The length of radius vector of a point (x, y) on the curve is xi yj + , that is,
2 2
. x y + It is given that
2 2 2
1 ( ') x y y y + = +
Squaring on both sides of this equation, we get
2 2 2 2
2
2 2 2 2
2
2
[1 ( ') ]
or
x y y y
dy
x y y y
dx
dy
x y
dx
dy dy
y x y x
dx dx
+ = +
| |
+ = +
|
\ .
| |
=
|
\ .
= =

Now,
dy
y x
dx
=

ydy xdx =

Integrating on both sides, we get
2 2
2 2 2 2
2 2
2 or Constant
y x
c
x y c x y
= +
= =

This answer does not exist in the given options. So, consider the other alternative:
y dy x dx =

Integrating on both sides, we get

2 2
2 2
y x
c = +
2 2
Constant x y + =
and this constant is > 0 in practical sense.
Correct Option: (2).
Question 22: Find the value of
/ 4
4
0
(sin ) . x dx
t
}

(1)
3
16
t
(2)
3 1
32 4
t

(3)
3 3
32 4
t
(4)
3 7
16 8
t

Chapter: Definite Integration
Level of Difficulty: Easy
Solution
We have
/ 4
4
0
(sin ) I x dx
t
=
}
(1)
We know that
2
1 cos 2
sin
2
x
x

=
Therefore,
2
4 4
2
1 cos 2
sin (sin )
2
1
[1 2cos 2 (cos 2 ) ]
4
1 1 cos 4
1 2cos 2
4 2
1 3 cos 4
2cos 2
4 2 2
x
x x
x x
x
x
x
x
| |
= =
|
\ .
= +
+ | |
= +
|
\ .
| |
= +
|
\ .

Substituting this value of
4
sin x in Eq. (1), we get

/ 4
0
/ 4
/ 4 / 4
0 0
0
3 1 1
cos 2 cos 4
8 2 8
3 1 1
[sin 2 ] [sin 4 ]
8 4 32
3 1 1
. (1 0) (0 0)
8 4 4 32
3 1
32 4
I x x dx
x x x
t
t
t t
t
t
| |
= +
|
\ .
(
= +
(

| |
= +
|
\ .
=
}

Correct Option: (2).
Alternate Solution: We have
/ 4
4
0
(sin ) I x dx
t
=
}

which can be written as
2 2
2 2 2
2
(sin ) (1 cos )
1
sin 4sin cos
4
1 cos 2 1
(sin 2 )
2 4
1 1 1 1 cos 4
sin 2
2 4 4 2
sin 2 1 1
sin 4
2 4 8 32
3 sin 2 sin 4
8 4 32
J x x dx
xdx x x dx
x
dx x dx
x
x x dx
x x
x x c
x x
x c
=
=

=
= + +
= + +
}
} }
} }
}


Using the given limits, the above equation becomes

/ 4 / 4 / 4
/ 4
0
0 0 0
3 sin 2 sin 4
[ ]
8 4 32
3 1
32 4
x x
I J x
t t t
t
t
( ( (
= = +
( ( (

=


Question 23: The area of the loop formed by
2 3
(1 ) y x x dx = is
(1)
1
4
0
x x dx
}

(2)
1
4
0
2 x x dx
}


(3)
1
4
1
x x dx


}

(4)
1/2
4
0
4 x x dx
}

Chapter: Area Under the Curve
Level of Difficulty: Moderate
Solution: We have
2 3
(1 ) y x x = (1)
For 1 x > ,
2
y is negative. Since the square of a real number cannot be negative, y does not exist at x =
0 or at x = 1; y = 0. Let
1
2
x = . Therefore, from Eq. (1), we get
2
1 1 7
1
2 8 16
7
4
y
y
| |
= =
|
\ .
=


Also, for x < 0,
2
y is negative. Therefore, the required area is
1 1
3
0 0
1
4
0
2 2 ( ) 1
2
y dx x x dx
x x dx
= +
=
} }
}

Correct Option: (2).
Quick Tip: When x < 0, no curve exists; when x < 1, no curve exists. Therefore, y
2
cannot be negative if y
is real.

1

Question 24: The equation of a plane passing through the line of intersection of the planes
2 10 0 x y z + + = and 3 5 x y z + = and passing through the origin is
(1) 5 3 0 x z + = (2) 5 3 0 x z =
(3) 5 4 3 0 x y z + + = (4) 5 4 3 0 x y z + =
Chapter: Three-Dimensional Geometry
Level of Difficulty: Moderate
Solution
We know that the equation of the plane passing through the line of intersection of planes
1
0 p = and
2
0 p = is
1 2
0 p p + =

That is,
( 2 10) (3 5) 0 x y z x y z + + + + =
(1)
Since, this plane passes through the origin, (0, 0, 0) satisfies this equation. This implies that
( 10) ( 5) 0 + =

2 =

Substituting the value of in Eq. (1), we get
( 2 10) 2(3 5) 0 x y z x y z + + + =

That is,
5 3 0
5 3 0
x z
x z
+ =
=

Correct Option: (2).

Question 25: Find the locus of a point whose distance from x-axis is twice the distance from the point
(1, 1, 2) .
(1)
2
2 2 4 6 0 y x y z + + = (2)
2
2 2 4 6 0 x x y z + + =
(3)
2
2 2 4 6 0 x x y z + + = (4)
2
2 2 4 6 0 z x y z + + =
Chapter: Three-Dimensional Geometry
Level of Difficulty: Hard
Solution: Let the point ( , , )
P
P P
P x y z be the required point.
The distance of the point from x-axis is
2 2
.
P P
y z +
The distance from the point (1, 1, 2) is

( )
2
2 2
( 1) 1 ( 2)
P P P
x y z + + +
2 2 2 2 2
( 1) ( 1) ( 2)
P P P P P
y z x y z + = + + +
2
2 2 4 6 0
P P P P
x x y z + + =
Therefore, the locus of point P is
2
2 2 4 6 0 x x y z + + =

Correct Option: (3).

Question 26: If the axes are rotated through 60 in the anticlockwise sense, find the transformed form of
the equation
2 2 2
x y a = .
(1)
2 2 2
3 3 2 X Y XY a + = (2)
2 2 2
X Y a + =
(3)
2 2 2
2 3 2 Y X XY a = (4)
2 2 2
2 3 2 X Y XY a + =
Chapter: Rectangular Coordinate System
Level of Difficulty: Moderate
Solution: Let ( , ) x y and ( , ) X Y be the old and the new coordinates, respectively. Since, the axes are
rotated in the anticlockwise direction, 60 . u = + Therefore,
cos60 sin60
sin60 cos60
x X
y Y
( ( (
=
( ( (


1 3
2 2
3 1
2 2
x X
y Y
(

(
( (
(
=
( (
(

(


3
2 2
3
2 2
X
Y
x
y
Y
X
(

(
(
(
=
(
(

+
(


3 3
and
2 2 2 2
x Y
x Y y X = = +
2 2
2
3 3
2 2 2 2
X y
Y X a
| | | |
+ =
| |
| |
\ . \ .

2 2 2 2
( 3 2 3 ) (3 2 3 ) X Y XY X Y XY yc + + + =
2 2 2
2 2 4 3 4 X Y XY a + =


2 2 2
2 3 2 Y X XY a =

which is the required equation
Correct Option: (3).

Question 27: The straight lines 3 4 0, 3 4 0 x y x y + = + = and 0 x y + = form a triangle which is
(1) equilateral (2) right-angled
(3) acute-angled and isosceles (4) obtuse-angled and isosceles
Chapter: Straight Lines and Pair of Lines
Level of Difficulty: Moderate
Solution: The following figure depicts the condition. By observation from the figure, ABC A is clearly an
obtuse angled and isosceles triangle.

Correct Option: (4).
Alternate Solution: The following figure depicts the condition.


From the figure, we get
A: 3 4 and 2; 2 x y y x x y + = = = =
B: (1,1) by solving the equations.
C: 3 4 0 and 2; 2 x y y x x y + = = = =
Thus,
2 2
1 9 10
4 4 4 2
10 10 16(2)
cos 0
2( 10)( 10)
AB BC
AC
B
= = + =
= + =
+
= <

Therefore, the given triangle is isosceles and obtuse angled triangle.


Question 28: Find the equation of the circle whose diameter is the common chord of the circles
2 2 2
( ) x a y a + = and
2 2 2
( ) x y b b + =
(1)
2 2 2 2
( )( ) 2 ( ) 0 x y a b ab bx ay + + + = (2)
2 2 2 2
( )( ) 2 ( ) 0 x y a b ab bx ay + + + + =
(3)
2 2 2 2
( )( ) 2 ( ) 0 x y a b ab bx ay + + = (4)
2 2 2 2
( )( ) 2 ( ) 0 x y a b ab ax by + + + =
Chapter: Circle
Level of Difficulty: Easy
Solution: Since
2 2 2
( ) x a y a + = , it is concluded that this circle has centre on x-axis and the circle passes
through origin.
Since
2 2 2
( ) x y b b + = , it is concluded that this circle has centre on y-axis and the circle passes through
origin.
Therefore,
2 2
2 0 (1) x y ax + =
2 2
2 0 (2) x y by + =
Subtracting Eq. (2) from Eq. (1), we get the common chord of the two circles:
(3) ax by
Solving Eqs. (1) and (3); we get the point of intersection of the two given circles:
2
2
2 0
a
x x ax
b
| |
+ =
|
\ .

2
2
0 or 2 0
a
x x x a
b
= + =
2
2 2
2
0or
ab
x x
a b
= =
+

2
2 2
2
0or
a b
y y
a b
= =
+

Therefore, the equation of circle having (0, 0) and
2 2
2 2 2 2
2 2
,
ab a b
a b a b
| |
|
|
+ +
\ .
as end points of one of its diameter
is
2 2
2 2 2 2
2 2
( 0) ( 0) 0
ab a b
x x y y
a b a b
| | | |
+ =
| |
| |
+ +
\ . \ .

2 2
2 2
2 2 2 2
2 2
0
ab a b
x x y y
a b a b
+ =
+ +


2 2 2 2
( )( ) 2 ( ) 0 x y a b ab bx ay + + + =
Correct Option: (1).

Question 29: Tangents are drawn from the point (4, 2) to the curve x
2
+ 9y
2
= 9. The angle between these
tangents is
(1)
3 3
5 17
(2)
43
10

(3)
43
5
(4)
3
17

Chapter: Ellipse
Level of Difficulty: Hard
Solution
The combined equation of pair of tangents drawn from a point (x
1
, y
1
) to the ellipse
2 2
2 2
1 0
x y
S
a b
+ = is
S
1
2
= S S
11
. Therefore,
2
2 2 2 2
1 1 1 1
2 2 2 2 2 2
2
2 2
2 2
2 2
2
1 1 1
4 4
2 1 1 2 1
9 9 9
3 7 16 8 36 52 0
2
tan
xx yy x y x y
a b a b a b
x x
y y
x y xy x y
h ab
a b
o
| | | |
| |
+ = + +
| |
|
| |
\ .
\ .\ .
| || |
| |
+ = + +
| |
|
| |
\ .
\ .\ .
+ + + =

=
+

where, a = 3, b = 7 and h = 8. Therefore,
2 64 21 43
tan
10 5
o

= =

Note: is acute angle between the pair of tangents. Therefore,
2 2 2 2
( ) 2 2 2 a b c a b c ab ac bc + = + + +
Correct Option: (2).
Quick Tip
- S
1
2
= S S
11

-
2
2
tan
h ab
a b
o

=
+
, that is,

1 2
1 2
2
1 2 1 2
1 2
tan
1
( ) 4
1
m m
m m
m m m m
m m
o

=
+
+
=
+


Alternate Solution: Any line passing through the point (4, 2) is given by
2 ( 4) y m x =
4 2 y mx m = +
For this line to be tangent to the given ellipse, put this y into the equation of the ellipse and make
D = 0
That is,
2
2
2 2 2 2
( 4 2) 1
9
(1 9 ) (36 72 ) 16(9) 16(9) 27 0
x
mx m
m x x m m m m
+ + =
+ + + + =

Now,
2
2 2 2 2
2 2 2 2
2 2 2 4 2
2
0 4 0
(36 72 ) 4(1 9 )(16.9 16.9 27) 0
(36 ) (1 2 ) 36(1 9 )(16 16 3) 0
(1 4 4 ) 36(16 16 3 9.16 9.16 27 ) 0
7 16 3 0
D B AC
m m m m m
m m m m m
m m m m m m m m
m m
= =
+ + =
+ + =
+ + + + =
+ =

Now,
2
1 2 1 2 1 2
1 2 1 2
2
2
( ) 4
tan
1 1
16 3
4.
7 2
tan
3
1
7
7 16 4.3.7
10 7
1
4(43)
10
43
5
m m m m m m
m m m m
o
o
+
= =
+ +
| |

|
\ .
=
+
| |

| =
|
\ .
| |
=
|
\ .
=

where is the acute angle between the tangents.


Question 30: The chord of contact of a point
A A
A( , ) x y of
2
4 y x = passes through (3, 1) and point A lies
on
2 2 2
5 x y + = . Then
(1)
2
A A
5 24 11 0 x x + + = (2)
2
A A
13 8 21 0 x x + =
(3)
2
A A
5 24 61 0 x x + + = (4)
2
A A
13 21 31 0 x x + =
Chapter: Parabola
Level of Difficulty: Easy
Solution: The following figure depicts the condition. Chord of contact of a point
A A
A( , ) x y with respect
to
2
4 y x = is
A A
2( ). y y x x = + Since this chord passes through the point (3, 1), we have
A A
2( 3) y x = +



AB and AC are tangents to the parabola.
BC is chord of contact of point A with respect to the parabola
2
4 . y ax =
Given that point A lies on
2 2
25 x y + = , we have

2 2
A A
2 2
A A
2 2
A A A
2
A A
2
A A
25
4( 3) 25
4( 9 6 ) 25
5 24 36 25 0
5 24 11 0
x y
x x
x x x
x x
x x
+ =
+ + =
+ + + =
+ + =
+ + =

Quick Tip: The chord of contact of point
A A
A( , ) x y with respect to
2
4 0 y ax = is
A A
2( )( ) 0 y y a x x + =
Correct Option: (1).



























Part B Physics


Question1. The figure below shows two different arrangements of materials 1, 2 and 3 to form a wall. All
the three slabs are of equal thickness. The thermal conductivities are
1 2 3
k k k > > . The left side of the wall
is 20
o
C higher than the right side. Which of the following statements is correct for steady state heat flow
across the wall?

(1) The temperature difference across 1 will be the greatest in both the arrangements.
(2) The rate of heat transfer across the wall will be the same for both the arrangements.
(3) The temperature difference across 1 will be greater in the first arrangement than in the second
arrangement.
(4) The rate of heat transfer across the wall will be greater in the first arrangement than in the second
arrangement.
Chapter: Conduction
Level of Difficulty: Moderate
Solution
In both the arrangements, the three materials are in series. Since, the temperature difference across the
wall is the same for both the arrangements, the heat current will also be the same for both. Hence, the rate
of heat transfer across the wall will be the same for both the arrangements.
Correct Option: (2)

Question2. A bubble from the tank of a scuba diver contains
4
3.5 10 mol

of gas. The bubble expands


as it rises to the surface from a freshwater depth of 10.3 m. Assuming that the gas is an ideal gas and the
temperature remains constant at 291 K, the amount of heat that flows into the bubble is
(1)
1
0.423 ln
2
| |
|
\ .
(2)
1
0.846 ln
2
| |
|
\ .


(3) 0.846 ln(2) (4) 0.423 ln(2)
Chapter: Thermodynamics
Level of Difficulty: Hard
Solution
The process of expansion of the gas is an isothermal process. Therefore, 0 U A = andQ W = .
2
1
ln
V
Q W nRT
V
| |
= =
|
\ .

Since the process is isothermal,
2 1
1 2
V P
V P
= ,
where
1
P is the pressure inside the bubble at the bottom and
2
P is the pressure at the top of the surface.
2
1 atm P = .

1 2
Pressure of 10.3 m column P P = + of water
= 1atm + 1 atm [since pressure of 10.3 m column of water = 1
atm]
2 atm =
Hence,
2 1
1 2
2
V P
V P
= =
So, the amount of heat that flows into the bubble will be

( )
( )
2
1
4
ln
3.5 10 8.31 291 ln 2
0.846 ln 2
Q W
V
nRT
V

=
| |
=
|
\ .
=
=

Correct Option: (3)

Question3. An ideal gas is taken along the cycle ABCA as shown in the figure given below with
1
40Pa P = ,
3
1
1.0m V = ,
2
10Pa P = , and
3
2
4.0m V = . The amount of heat transferred into the system
during the entire cyclic process is


(1) 45 J (2) 45 J
(3) 55 J (4) 55 J
Chapter: Thermodynamics
Level of Difficulty: Easy
Solution
For the cyclic process, change in internal energy 0 U A = . So, using first law of thermodynamics we get
Q W =
Now, the work done will be
W = the area enclosed inside the cycle

1 2 2 1
3 3
1
( )( )
2
1
(40Pa 10Pa)(4.0m 1.0m )
2
45 J
P P V V =
=
=

Thus, heat is transferred into the system during the entire cyclic process will be 45J. Q=
Correct Option: (2)

Question4. A ball suspended by a light inextensible string from a ceiling is shifted by an angle
0
u (as
shown in the figure below) so as to strike the vertical wall normally with the coefficient of restitution e.
The maximum angle made by the string with vertical after the first collision is

u0
O


(1)
{ }
1 2
0
cos 1 (1 cos ) e u

(2)
{ }
1 2
0
sin 1 (1 sin ) e u


(3)
0
u (4)
{ }
1 2
0
cos (1 cos ) e u



Chapter: Collisions
Level of Difficulty: Hard
Solution

u0
O
h1
l

Let the mass of ball be m and length of the string suspending the ball be l. Then the height h
1
with respect
to the position at collision is given by
( )
0 1
1 cos h l u =
Let v
1
be the speed of the ball before the first collision. Conserving mechanical energy up to first collision

i i f f
K U K U + = +

2
1 1
1
0 0
2
mgh mv + = +
( )
0
2
1
1
1 cos
2
gl v u =
( )
0 1
2 1 cos v gl u =
If v
2
is the speed of the ball just after the collision, then
( )
2 1 0
2 1 cos v ev e gl u = =
If ball rises to height h
2
after the collision then the angleu made by string with the vertical is related to it
as
( )
2
1 cos h l u =
Conserving mechanical energy after the first collision up to rise to height h
2


i i f f
K U K U + = +

2
2 2
1
0 0
2
mv mgh + = +
( ) ( )
0
2
1
2 1 cos 1 cos
2
e gl gl u u ( =



( ) ( )
0
2
1 cos 1 cos e u u =
( )
2
0
cos 1 1 cos e u u =
( ) { }
1
0
2
cos 1 1 cos e u u

=
Correct Option: (1)
Quick Tip: By substituting e = 1, you should get u
0
and by putting e = 0, you should get 0. That you get
only with
{ }
1 2
0
cos 1 (1 cos ) e u

and
{ }
1 2
0
sin 1 (1 sin ) e u

and no other option. By realizing the


relation of height with angular deviation, that is, ( ) 1 cos h l u = , you can zero in the right option of
{ }
1 2
0
cos 1 (1 cos ) e u

.

Question5. Planet A has mass
24
3.00 10 kg M = and radius
7
2.00 10 m R = , and it completes a full
rotation in time 35.0h T = . What is the free-fall acceleration g on its equator? (Take G =
11 2 2
6.67 10 Nm / kg

)
(1)
2
0.500ms

(2)
2
0.050ms


(3)
2
0.450ms

(4)
2
0.550ms


Chapter: Gravitation
Level of Difficulty: Moderate
Solution
The gravitational acceleration on the planets surface is given by

g 2
11 24
7 2
2
(6.67 10 )(3.00 10 )
(2.00 10 )
0.500ms
GM
a
R

=

=

=

The centripetal acceleration of any mass on the planets surface at the equator is

2
c
2
2 7
2
2
2
4 (2.00 10 )
(35 3600)
0.0497ms
a R
R
T
e
t
t

=
| |
=
|
\ .

=


In rotational frame any mass will experience a centrifugal force (= ma
c
) which is directed away from the
center. Hence, a
c
has to be subtracted from a
g
. Free-fall acceleration at the equator of the planet will be

g c
2 2
2
0.500ms 0.0497ms
0.450ms
g a a

=
=
=

Correct Option: (3)

Question6. The position of a particle moving along the x-axis is given by
3
7.8 9.2 2.1 x t t = + , with x in
meters and t in seconds. What is the velocity at 3s t = ?
(1)
1
21 ms

(2)
1
35 ms


(3)
1
47.5 ms

(4)
1
54ms


Chapter: Motion along a Straight Line
Level of Difficulty: Easy
Solution
Position of a particle moving along the x-axis is
3
7.8 9.2 2.1 x t t = +
The velocity of the particle will be

( )
3
2
7.8 9.2 2.1
9.2 6.3
dx
v
dt
d
t t
dt
t
=
= +
=

At 3s t = , the velocity of the particle is
2 1
9.2 6.3(3 ) 47.5 ms v

= =
Correct Option: (3)

Question7. A particle is launched into projectile motion over level ground. At point A, 3.0mabove
ground level, the particle has velocity
1 1

(5.0 ms ) ( 6.0 ms ) v i j

= + . What was its angle of launch?
(1)
1
6.25
tan
5

| |
|
\ .
(2)
1
3.65
tan
5

| |
|
\ .

(3)
1
8.59
tan
5

| |
|
\ .
(4)
1
9.74
tan
5

| |
|
\ .

Chapter: Motion in Two Dimensions and Projectile Motion

Level of Difficulty: Moderate
Solution
At h =3.0mabove ground level,
1 1

(5.0 ms ) ( 6.0 ms ) v i j

= +
Let the initial velocity be

x y
u u i u j = + .
Then
1
5.0 ms
x
u

= because for the projectile, the horizontal or x-component of velocity remains constant.
Using third kinematics relation along the vertical direction, we get

2 2
1
( 6) 2(9.8)(3.0)
9.74 ms
y
y
u
u

=
=

The required angle of launch will be
1 1
9.74
tan tan
5
y
x
u
u
u

| |
| |
= =
| |
\ .
\ .

Correct Option: (4)

Question8. A flat body of mass 2.00 kgis sliding on a frictionless surface, as two constant forces act on
it:
1

(10N) F j = and
2

( 4N) F j = . At 0 t = , the velocity of the body is


1
0

(4.00 ms ) v i

= . What is the
angle of the bodys velocity (relative to the positive x-direction) at 3.00s t = ?
(1)
1
3
tan
2

| |
|
\ .
(2)
1
3
tan
4

| |
|
\ .

(3)
1
9
tan
4

| |
|
\ .
(4)
1
5
tan
8

| |
|
\ .

Chapter: Newtons Laws of Motion
Level of Difficulty: Moderate
Solution
The acceleration of the body will be
1 2
2

10 4
2.00

(3.00 ) ms
F F
a
m
j j
j

+
=

=
=

So, to find out the velocity at 3.00s t = , we can write

0

4.00 (3.00 )(3.00)

4.00 9.00
v v at
i j
i j
= +
= +
= +

Thus the components of velocity are
1
4.00 ms
x
v

= and
1
9.00 ms .
y
v

=
If u be the angle made by this velocity with the positive x-direction, then
1
tan
9.00
4.00
9
tan
4
y
x
v
v
u
u

=
=
| |
=
|
\ .


Correct Option: (3)

Question9. A body is rotating around a fixed axis with angular velocity 3 rad s
1
with constant angular
acceleration of 1 rad s
2
at some time. Find the magnitude of acceleration of a particle 5 m away from the
axis after the body has turned by 90.
(1) ( )
2
2
5 1 9 ms t

+ + (2) ( )
2
5 9 ms t

+
(3)
2
5 ms

(4)
2
5 82 ms


Chapter: Rotational Kinematics
Level of Difficulty: Moderate
Solution
Initial angular velocity
0
e = 3 rad s
1
.
Angular accelerationo = 1 rad s
2
.
If angular velocity after turning by u A = 90 ise , then we have,

2 2
0
2 e e o u = + A

2
2
9 2 1
2
9
t
e
e t
= +
= +

Acceleration will have two components, tangential and centripetal. The centripetal component of
acceleration is
( )
2
c
2
9 5 ms r a e t

= + =
and the tangential component is


2
t
5 1 ms a ro

= =
The net acceleration will be
( )
( )
2 2
c t
2
2
2
2
5 9 1
5 1 9 ms
a a a
t
t

= +
= + +
= + +

Correct Option: (1)

Question10. Two boxes, A and B, are connected to each other as shown in the figure given below.

The system is released from rest and the box B falls through a distance of 1.00m. The surface of the table
is frictionless. What is the kinetic energy of box B just before it reaches the floor?
(1) 2.45J (2) 4.90J
(3) 9.80J (4) 29.4J
Chapter: Work and Energy
Level of Difficulty: Moderate
Solution
Let T be the tension in the string as shown in the figure given below.


We can write the force equation for both the masses as:
B B
A
(1)
(2)
m g T m a
T m a
=
=

Adding Eq. (1) and Eq. (2), we get
B B A
B
A B
( )
m g m a m a
m
a g
m m
= +
=
+

Substituting value of a in Eq. (2), we get
A B
A B
( )
3.00 1.00
3.00 1.00
3.00
4.00
m m
T g
m m
g
g
=
+
| |
=
|
+
\ .
| |
=
|
\ .

Since the system is released from rest, initial kinetic energy
i
0J. K =
Final kinetic energy of box B is equal to the net work done on it by the tension force and the gravitational
force.
f
B
B
2
( ) (1.00)
3.00
1.00 (1.00) [where 9.8ms ]
4.00
9.8
4
2.45 J
K W
Fd
m ad
m g T
g g g

=
=
=
=
| |
= =
|
\ .
=
=

Correct Option: (1)

Question11. Two uniform steel bars are freely hanging from a ceiling. The length of the second bar is
double that of the first. The ratio of elongation of first bar to that of second is
(1) 1 : 4 (2) 4 : 1
(3) 2 : 1 (4) 1 : 2
Chapter: Elasticity
Level of Difficulty: Moderate
Solution

Let the first bar be of mass M, length L, cross section A, mass per unit length m and Youngs modulus Y.
Therefore,
M = L
Elongation of segment dx, at a distance x from the free bottom is given by
dx
dx Tx
AY
dx
mxg
AY
Mx dx
g
L AY
Lx dx
g
L AY
dx
xg
AY

A =
=
=
=
=

Elongation of a freely hanging body
0
0
2
0
2
2
2
L
L
L
x dx
dx
xg
AY
g
xdx
AY
g x
AY
gL
AY

A = A
=
=
=
=
}
}
}

Therefore,
2
x L A
Hence, if length is doubled, elongation will quadruple.
Correct Option: (1)

Question12. The height of liquid in a capillary tube depends upon
(1) the outer radius of the tube. (2) the inner radius of the tube.
(3) the temperature of the tube. (4) the length of the tube.

Chapter: Surface Tension and Viscosity
Level of Difficulty: Easy
Solution
Rise of liquid in a capillary tube is given by the relation

2 cos T
h
r g
u

=
where r = inner radius of tube, T = surface tension and = density of liquid. Thus, height of liquid
depends on the inner radius of the capillary tube not the outer radius.
Correct Option: (2)

Question13. A particle executes SHM along x-axis about the center at x = a with frequency f. Initially
the particle is at rest at origin. Its equation of motion is
(1) (cos2 1) x a ft t = (2) (1 cos2 ) x a ft t =
(3) 2 (cos2 1) x a ft t = (4) 2 (1 cos2 ) x a ft t =
Chapter: Simple Harmonic Motion
Level of Difficulty: Moderate
Solution
About x = 0, SHM equation is
( ) sin 2 f x a t t | = +
For particle initially at rest at maximum positive displacement,
2
t
| = .
( ) sin 2 cos 2
2
x a t a t f f
t
t t
|
=
|
|
\ .
= + (1)
Shifting axis of SHM to x = a, Eq. (1) would be
( )
( )
cos 2
cos(2 )
os 2 1 c
x
f x a t
t f
a a t
a
f
a
t
t
t

=
+
=
=


Correct Option: (1)


Question14. A spherical source of radius 1 m emits sound equally in all directions in a non-dispersive
medium. The ratio of the magnitude of the amplitudes of the waves at two points at a distance of 25 m
and 36 m from the source is
(1) 36:25 (2) 5:6
(3) 6:5 (4) 216:125
Chapter: Sound Waves
Level of Difficulty: Moderate

Solution
Power transfer across a spherical surface with the same center as the source would be same across all such
surfaces by conservation of energy. Intensity at any point at a distance of r from the center, if power
generated by the source is P, would be
2
4
P
I
r t
=
Since
2
I A when other independent parameters remain same, so we can write

2
2
1
A
r


1
A
r

Hence, ratio would be 36:25 because amplitude is inversely proportional to distance.
Correct Option: (1)


Question15. Two 3.25 Fcapacitors are connected in series across the terminals of an 50.0Hz
alternating current (AC) generator that has a peak voltage of 244 V. What is the rms current in the
circuit?
(1) 0.124A (2) 0.088A
(3) 0.308A (4) 0.248A
Chapter: Alternating Current
Level of Difficulty: Moderate
Solution
Since the capacitors are in series, the effective capacitance of the circuit will be
(3.25)(3.25)
1.625F
(3.25 3.25)
C = =
+

Hence, capacitive reactance will be
6
3
1
2
1
2 (50.0)(1.625 10 )
1.96 10
C
X
fC t
t

=
=

= O

Peak current will be,


0
0
3
244
1.96 10
0.124 A
C
V
I
X
=
=

=

Therefore, the rms current will be
0
rms
2
0.124
2
0.088 A
I
I =
=
=


Correct Option: (2)

Question16. A stream of electrons is going round a circle in the counter clockwise direction as shown in
the figure below.

At the center of the circle, they produce a magnetic field that is directed
(1) into the page. (2) out of the page.
(3) to the left. (4) to the right.
Chapter: Magnetic Field due to a Current
Level of Difficulty: Easy
Solution
The direction of the conventional current is opposite to the direction of the flow of the electrons. So,
applying the right hand rule, we conclude that the magnetic field at the center will be directed into the
page.
Correct Option: (1)


Question17. In Youngs double slit experiment, blue light ( 440 nm) = gives a second order bright
fringe at a certain location on a flat screen. What wavelength of visible light would produce a dark fringe
at the same location? Assume that the range of visible wavelengths extends from 380 nm to 750 nm.
(1) 467 nm (2) 587 nm
(3) 671 nm (4) 736 nm
Chapter: Light Waves
Level of Difficulty: Moderate
Solution
Since it is a second-order bright fringe, we have
2
2 2(440) 880 D D D
y
d d d

= = =
where d is separation between the slits and D is distance of screen from slits.
For getting a dark fringe at the same position,

( 1/ 2) 880
1
880 (1)
2
m D D
d d
m

+
=
| |
+ =
|
\ .

where 0, 1, 2,... m=
For 0 m= , using Eq. (1) we get
1
0 880
2
1760 nm

| |
+ =
|
\ .
=

For 1 m= , using Eq. (1) we get
1
1 880
2
587 nm

| |
+ =
|
\ .
=
,
and for 2 m= , using Eq. (1) we get
1
2 880
2
352 nm

| |
+ =
|
\ .
=

So, the only value that falls within the given range of 380 nm to 750 nmis 587 nm. =
Correct Option: (2)


Question18. A diverging lens of focal length 14.0 cmis 30.0 cm to the left of a converging lens of focal
length12.0 cm. An object is placed 16.0 cm to the left of the diverging lens. How far is the final image
from the converging lens (after refraction through the converging lens)?
(1) 7.5 cm (2) 15.7 cm
(3) 17.6 cm (4) 27.4 cm
Chapter: Refraction
Level of Difficulty: Hard
Solution


Applying lens formula for the diverging lens,

1 1 1
1 1 1
1 1
16.0 14.0
15
112
7.47 cm
v u f
v u f
v
=
= +
= +

=
=

This image becomes the virtual object for the second (converging) lens. Thus, the object distance for the
converging lens = 7.47 30.0 37.47 cm. + =
We can now apply the lens formula to the converging lens.



1 1 1
1 1 1
1 1
37.47 12.0
37.47 12.0
25.47
17.6 cm
v u f
v u f
v
=
= +
= +

=

Thus, the final image will be formed 17.6 cm to the right of the converging lens.
Correct Option: (3)

Question19. If (n + 1) divisions on the vernier scale (VSD) of a vernier calipers coincide with n divisions
on the main scale (MSD) and each division on the main scale is of c units, then the least count of
instrument is
(1)
1
c
n +
(2)
2
c
n +

(3)
c
n
(4)
1
c
n

Chapter: Experimental Skills
Level of Difficulty: Moderate
Solution
Least count (LC) = 1 MSD 1 VSD
In the given question
(n) MSD (n + 1) VSD
1VSD MSD
1
n
n
| |
=
|
+
\ .

So, the least count will be
LC = I MSD MSD
1
n
n
| |

|
+
\ .

1
MSD
1
1
MSD
1
n n
n
n
+ | |
=
|
+
\ .
| |
=
|
+
\ .

Since each division on the main scale is of c units

LC
1
c
n
=
+


Correct Option: (1)

Question20. A wire of cross-sectional radius a is bent to form a circle of radius R (R >> a). A charge Q is
uniformly distributed on the ring. An infinite line of positive charges of linear charge density passes
through center of the ring, perpendicular to the plane of the ring. If Youngs modulus of the material of
the wire is Y, then the change in length of the wire after placing the infinite line of charge at the center of
charged ring is
(1)
2 2
0
2
Q
a Y

t c
(2)
2 2
0
Q
a Y

t c

(3)
2 2
0
2 Q
a Y

t c
(4)
2 2
0
3
2
Q
a Y

t c

Chapter: Electric Field
Level of Difficulty: Hard
Solution
Consider an infinitesimal element of the circle, as shown in the figure below.

Charge on the infinitesimal element =
2
Q
dx
R t

Electric field due to infinite line of charges at distance R is given by
0
2
E
R

tc
=
0
2 sin
2 2
Q
T dx
R R

u
tc t
=
0
(2 )
2 2
Q
T dx
R R

u
tc t
=

Since u is small
0
2 2
dx Q
T dx
R R R

tc t
| |
=
|
\ .

2
0
4
Q
T
R

t c
=
2
/
/
T a
Y
L L
t
=
A

2
2 T R
L
a Y
t
t

= A
2 2
0
2
Q
L
a Y

t c
A =

Correct Option: (1)

Question21. Read the two given statements S1 and S2 carefully and mark the correct options
S1: The potential difference between the shells of two concentric charged spherical shells depends on
charge of inner shell.
S2: Potential due to charge of outer shell remains same at every point inside the sphere.
(1) S1 and S2 are True and S2 is the correct explanation for S1.
(2) S1 is True and S2 is True but S2 is NOT a correct explanation for S1.
(3) S1 is True and S2 is False.
(4) S1 is False and S2 is True.
Chapter: Electric Potential
Level of Difficulty: Easy
Solution
Consider the two concentric shells A and B shown in the figure below.

Potential due to shell A

1 2
A
0 2
1
4
Q Q
V
R tc
| | +
=
|
\ .

Potential due to shell B
1 2
B
0 1 2
1
4
Q Q
V
R R tc
| |
= +
|
\ .

The potential difference will be
B A 1
0 1 2
1 1 1
4
V V V Q
R R tc
| |
A = =
|
\ .


Hence the potential difference between the shells of two concentric charged spherical shells depends on
charge of inner shell because potential due to charge of outer shell remains the same at every point inside
the sphere.
Correct Option: (1)

Question22. A conducting wire of length 1 m, being used as a potentiometer wire, has radius linearly
changing from r to 2r across its ends. The distance of the null point from the end with radius r, if the
resistances in the gaps are in the ratio 2 : 3, is
(1) 25 cm (2) 40 cm
(3) 20 cm (4) 33.33 cm
Chapter: Electric Current in Conductors
Level of Difficulty: Hard
Solution
Since the radius of the wire increases linearly with distance x from left end, we have the radius R at a
distance x as
R = r +
xr
l

The resistance of an element dx at a distance x is
2 2
dx dx
r
R
xr
r
l

t t
A = =
| |
+
|
\ .



If the null point occurs at a distance x from the left end, we have the ratio of the two parts (left and
right) in the ratio 2: 3. This implies

2
0
2
2
3
x
x
dx
xr
r
l
dx
xr
r
l
| |
+
|
\ .
=
| |
+
|
\ .
}
}

3
2
0
( )
x
dx
l x
=
+
}
2
2
( )
l
x
dx
l x +
}

3
0
1 1
2
x
x l
l x l x
( (
=
( (
+ +


3
1 1 1 1
2
2 l l x l x l
| | | |
=
| |
+ +
\ . \ .

Substituting l = 100 cm and simplifying above expression, we get

1 1 1 1
3 2
100 100 100 200 x x
| | | |
=
| |
+ +
\ . \ .

x = 25 cm
Correct Option: (1)

Question23. When a shunt of resistance 2O is connected in parallel to galvanometer, its range to measure
current becomes 11 times. Then the resistance required to be connected in series to convert it into
voltmeter of range 20 times is
(1) 380 O (2) 280 O
(3) 180 O (4) 480 O
Chapter: Electric Current in Conductors
Level of Difficulty: Easy
Solution
Now the shunt S can be expressed as

g
g
1
R
S
I
I
=



g
2
11 1
R
=


g
20 R = O
The required resistance can be calculated using
g
g
1
V
R R
V
| |
= |
|
\ .

20 (20 1)
380
=
= O


Correct Option: (1)

Question24. The graph given below, plots photocurrent versus anode potential for a cathode with 4eV
work function. The energy of the incident photon is

(1) 6 eV (2) 4 eV
(3) 2 eV (4) 8 eV
Chapter: Photoelectric effect
Level of Difficulty: Easy
Solution
Given | = 4 eV
From the graph, stopping potential V
0
= 2V. This implies that
K
max
= eV
0
= 2 eV
But from Einsteins photoelectric equation
hv = K
max
+ |
hv = (4 + 2 )eV = 6eV

Correct Option: (1)


Question25. An inductance L and a resistance R are connected in series to an ideal battery. A switch in
the circuit is closed at time 0, at which time the current is zero. The rate of increase of the energy stored in
the inductor is maximum
(1) just after the switch is closed.
(2) at time / t L R = after the switch is closed.
(3) at time 2 / t L R = after the switch is closed.
(4) at time ( / ) ln2 t L R = after the switch is closed.
Chapter: Inductor
Level of Difficulty: Hard
Solution
The transient current in an LR circuit is given by
1 exp
/
V t
I
R L R
( | |
=
| (
\ .


Therefore,
exp
/
dI V t
dt L L R
| |
=
|
\ .

Energy stored in the inductor is given by
2
1
2
E LI =
Rate of change of energy,
2
exp 1 exp
/ /
dE di V t t
LI
dt dt R L R L R
( | | | |
= =
| | (
\ . \ .


2 2
2
exp 1 2exp
/ /
d E V t t
dt L L R L R
( | | | |
=
| | (
\ . \ .

Now, for
dE
dt
to be maximum,
2
2
0
d E
dt
=
Thus,
2
exp 1 2exp 0
/ /
V t t
L L R L R
( | | | |
=
| | (
\ . \ .

1
exp
/ 2
ln 2
/
( / ) ln 2
t
L R
t
L R
t L R
| |
=
|
\ .
=
=


Correct Option: (4)

Question26. For a logic gate, the voltage waveforms A and B are shown as input in the figure given
below and C as output. The logic circuit gate is



(1) OR gate
(2) AND gate
(3) NAND gate
(4) NOR gate

Chapter: Digital Electronics
Level of Difficulty: Moderate
Solution
The truth table corresponding to waveforms is as shown here. It indicates that the given logic circuit gate
is AND gate.

A B C
0 0 0
1 0 0
0 1 0
1 1 1


Hence, the correct option is (2).

Correct Option: (2)

Question27. The atomic mass of
10
B is 10.811 u. The binding energy of
10
B nucleus is (given that the
mass of electron is 0.0005498 u, the mass of proton is m
p
= 1.007276 u, and the mass of neutron is m
n
=
1.008665 u)

(1) 681.201 MeV (2) 678.932 MeV

(3) 378.932 MeV (4) None of these
Chapter: Atomic Nucleus
Level of Difficulty: Moderate
Solution
Mass of
10
B nucleus is M = 10.811u
Mass of proton m
p
= 1.007276 u
Mass of neutron m
n
= 1.008665u
The number of protons and neutrons in
10
B nucleus is 5 each.
The mass defect will be

1 007276 5 1 008665 10 811
5 03638 5 043325 10 811
10 079705 10 811
0 731295u
5 5
5
p n
m
. . .
. . .
.
m
.
m
.
M
+
= +
A =
=
=
+
=

The binding energy will be

( )
( )
2
931.5 MeV
0.731295 931.5 MeV
681.201MeV
E mc
m
= A
= A
=
=

Correct Option: (1)


Question28. In case of p-n junction diode at high value of reverse bias, the current rises sharply. The
value of reverse bias is known as
(1) cut-off voltage.
(2) Zener voltage.
(3) inverse voltage.
(4) critical voltage.



Chapter: Semiconductors
Level of Difficulty: Easy
Solution
In case of p-n junction diode at high value of reverse bias, the current rises sharply. The value of reverse
bias is known as Zener voltage.
The Zener diode or breakdown diode, as it is sometimes called, is basically the same as the
standard p-n junction diode but is specially designed to have a low predetermined reverse breakdown
voltage that takes advantage of this high reverse voltage. The Zener diode is the simplest types of voltage
regulator and the point at which a Zener diode breaks down or conducts is called the Zener voltage.
Correct Option: (2)

Question29. A T.V. transmission tower has a height of 180 m. The broadcast is available up to a distance
of (Radius of Earth ~ 6.4 10
6
m)
(1) 48 km (2) 24 km
(3) 34 km (4) 68 km
Chapter: Communication Systems
Level of Difficulty: Easy
Solution
The range of broadcast can be calculated using relation
6
3
2
2 180 6.4 10
48 10 m
48km
d hR =
=
=
=

Correct Option: (1)

Question30. Read the two statements S1 and S2 are given below and answer the correct option
S1: A normal human eye can clearly see all the objects just beyond a certain minimum distance.
S2: The human eye has the capacity to adjust suitably the focal length of its lens to a certain extent.
(1) S1 is True, S2 is True and S2 is a correct explanation for S1.
(2) S1 is True, S2 is True but S2 is NOT a correct explanation for S1.
(3) S1 is True but S2 is False.
(4) S1 is False but S2 is True.
Chapter: Optical Instruments


Level of Difficulty: Moderate
Solution
A normal human eye can clearly see all the objects just beyond a certain minimum distance which is 25
cm for normal eye and it is known as near point. Hence statement 1 is correct.
Accommodation is the adjustment to the thickness of the lens in the eye to ensure that the image on the
retina is sharp. When thickness of the lens changes the focal length of the lens also changes. Thus, the
human eye has the capacity to adjust suitably the focal length of its lens to a certain extent. Therefore
statement 2 is also correct.
Statement 1 and statement 2 are two independent statements. Therefore, statement 2 is not a correct
explanation of statement 1.
Correct Option: (2)



























Part C Chemistry


Question 1: Which of the following reactions does not contribute to air pollution in the troposphere?
(1)
4 10 2 2 2
C H (g) + 6O (g) CO(g) + 2CO (g) + 5H O(l)
(2)
Lightning
2 2
2 2
N (g) O (g) 2NO
2NO(g) + O (g) 2NO (g)
+


(3)
4 2
2HCHO CH (g) + CO (g)
(4)
3 2
Cl + O (g) ClO + O (g)
Chapter: Environmental Chemistry
Level of Difficulty: Moderate
Solution
The first reaction takes place during incomplete combustion of fossil fuels and releases oxides of carbon
into the troposphere. The second set of reactions represents formation of oxides of nitrogen in the
troposphere. The third reaction represents formation of hydrocarbons due to incomplete combustion of
fuels or anaerobic decomposition of organic matter. The fourth reaction between chlorine radical and
ozone takes place in the stratosphere and leads to depletion of ozone in the stratosphere. Chlorine free
radical is obtained from chlorofluorocarbons released into the atmosphere.
2 2 2
F CCl (g) F C Cl(g) + Cl(g)
hv

Correct Option: (4)

Question 2: The four elements Ca, Mg, P and Cl are to be arranged in the order of their increasing
atomic sizes. Which of the following orders is correct?
(1) Cl < P < Mg < Ca (2) P < Cl < Ca < Mg
(3) Ca < Mg < Cl < P (4) Mg < Ca < P < Cl
Chapter: Classification of Elements and Periodicity of Properties
Level of Difficulty: Moderate
Solution


Element Atomic
number
Number of
electrons
Cl 17 17
P 15 15
Mg 12 12
Ca 20 20
As the Z
eff
increases along a period, the atomic radius decreases, so the order is Mg < P < Cl. Down the
group, there is increase in the number of shells, hence the radius increases. So, the order is Mg < Ca.
Correct Option: (4)

Question 3: During the concentration of sulphide ores by froth floatation process, the separation of
sphalerite and galena is achieved by which of the following substances used as depressant?
(1) Potassium xanthate (2) Sodium cyanide
(3) Copper sulphate (4) Pine oil
Chapter: General Principles and Processes of Isolation of Elements
Level of Difficulty: Moderate
Solution
Collector, that is, xanthate acts only on solid particles through adsorption. The two ores show different
reactivity towards NaCN:

2 2 4
(Soluble complex)
ZnS 4NaCN Na S Na Zn(CN)
PbS NaCN No reaction
+ +
+

Correct Option: (2)

Question 4: Which one of the following is an electron-deficient species?
(1) B
2
H
6
(2) NH
3
(3)
4
AlH


(4) CH
4

Chapter: p-Block Elements
Level of Difficulty: Easy
Solution
After having 3 center2 electron bond also, B
2
H
6
is still electron deficient which can be evidenced by the
following spontaneous reaction:

2 6 2 6
B H 2THF B H THF +


The octet/duplet is complete for the central atom in the rest of the hydrides, so they are not electron
deficient.
B
2
H
6
:

NH
3
:
4
AlH

: CH
4
:

Correct Option: (1)

Question 5: When spherically symmetrical field of ligands surrounds the central metal ion, then which of
the following options is correct for the change in energy order of d-orbitals of the central metal ion?
(1) t
2g
> e
g
(2) e
g
> t
2g
(3) e
g
= t
2g
(4) Cannot be predicted
Chapter: Coordination Compounds
Level of Difficulty: Moderate
Solution
If a symmetrical field of negative charges surrounds a metal ion, the d-orbitals remain degenerate.
Correct Option: (3)

Question 6: Which of the following pair of elements of lanthanoids do not follow the regular trend of
decrease in radius from Ce to Lu?
(1) Eu and Gd (2) Sm and Tm (3) Pr and Er (4) Eu and Yb
Chapter: d and f-Block Elements
Level of Difficulty: Moderate
Solution
The number of electrons participating in the metallic bonding is three for all the lanthanoid elements
except Eu (At. no. 63) and Yb (At. no. 70) as the outermost electrons are favored to participate in
bonding. In these two cases, one electron from 5d orbital is shifted to 4f orbital to get half-filled and
fully filled stable electronic configuration. Hence, these two elements use only two electrons in metallic
bonding, and atomic radii do not show the expected decrease.
Correct Option: (4)




Question 7: Identify the correct designation for this tripeptide:

(1) Cys-Met-Glu (2) Cys-Met-Asp (3) Met-Cys-Glu (4) Asp-Cys-Met
Chapter: Biomolecules
Level of Difficulty: Easy
Solution
Based on the respective structures of amino acids, the correct designation is Cys-Met-Glu.
Correct Option: (1)

Question 8: Which of the following ketones has the largest equilibrium constant for the addition of
water?
(1) (2)
(3) (4)
Chapter: Aldehydes and Ketones
Level of Difficulty: Hard
Solution
The equilibrium constant for hydration depends on stability of produced hydrate and ketone. Since the
stability of ketone increases with the increasing electron density on carbonyl carbon atom, its stability is
increased by electron donating group.
In contrast, the hydrates stability is increased by electron withdrawing group, but stability of ketone is
decreased.


Since equilibrium constant is the ratio of concentration of product to reactant, it would be more if
product concentration is more. If product is more stable than reactant then equilibrium constant will be
more.
Therefore, equilibrium constant would be the maximum in the case of most stable hydrate formed.
Option (3) contains ketone with the most electron withdrawing group. This ketone is least stable and its
hydrate is most stable, so it has largest equilibrium constant for addition of water.
Correct Option: (3).
Quick Tip: Just look for most stable hydrate formed. Stability of hydrate increases by electron
withdrawing group, so, compound which contains most powerful electron withdrawing group would
have largest equilibrium constant.

Question 9: Which of the following ketones could not be prepared by an acid catalyzed hydration of an
alkyne?
(1) (2)
(3) (4)
Chapter: Hydrocarbons: Alkynes
Level of Difficulty: Easy
Solution
Alkynes on reacting with water in the presence of water initially form an enol, which on tautomerization
gives ketone. So, in this structure we have to check whether an alkyne is possible for the given structure
of ketone.
3
H O Tautomerization
Alkynes Enol Ketone
+

H
2 2
| |
| |
R C C R H O R C C R R CH C R
H OH
O
+
+ =
For the given ketones, all alkynes should be internal alkynes. Overall change during this reaction is that
triple bond converts into single bond and one double bond appears at one carbon of triple bond and
two hydrogen atoms attached with second carbon atom of triple bond. So, alkynes can be drawn by


placing triple bond at carbonyl carbon. In this way, we cannot place a triple bond in the compound given
in option (3). So, this ketone cannot be formed by alkynes on hydration.
Correct Option: (3)
Quick Tip: Just look for minimum two hydrogen atoms at alpha position. If they exist, then alkyne
formation is possible with the same skeleton and we can also form a ketone from it.

Question 10: Potassium phthalimide on reaction with compound (A) followed by hydrolysis forms
isopentyl amine. The compound A will be
(1)
3 3 3
CH CH(CH ) CH(Br) CH (2)
3 3 2 3
CH C(CH )(Br) CH CH
(3)
2 3 2 3
Br CH CH(CH ) CH CH (4)
3 3 2 2
CH CH(CH ) CH CH Br
Chapter: Alkyl and Aryl halides
Level of Difficulty: Moderate
Solution
The given reaction sequence is for Gabriel phthalimide synthesis. The synthesis is successful only if
haloalkane is primary. Secondary haloalkanes produce elimination product, alkenes.

RNH
2
is iso-pentyl amine,

So A will be

Correct Option: (4)
Quick Tip: The NH
2
group will be attached to that carbon atom at which B atom is present.

Question 11: Which of the following molecules would you expect to be aromatic?
NK + RBr
O
O
+ H O/H
2
+
KBr
C OH
C OH
O
O
+ RNH
2
(A)
CH
3
CH CH
2
CH
2
NH
2
CH
3
CH
3
CH CH
2
CH
2
Br
CH
3



(1) I, II and III only (2) I, II, IV only (3) II, III, IV only (4) All are aromatic
Chapter: Hydrocarbons: Aromatic
Level of Difficulty: Moderate
Solution
According to Huckels rule, two conditions should satisfy for aromaticity. The planar cyclic compound
with (4n + 2) pi electrons are aromatic compounds.
I. This carbon ring contains 6 pi electrons with all carbon atoms in sp
2
hybridized state. The angle strain
in this ring is not much so it is also planar and aromatic.
II. Though nitrogen atoms can avail lone pairs for aromaticity, but in this case it is not required. Ring
contains 6 pi electrons and all atoms in sp
2

hybridized state. So, the compound is aromatic.
III. It contains 8 pi electrons, so it is not an aromatic compound.
IV. Ring contains 6 pi electrons with all carbon atoms sp
2
hybridized, so it is an aromatic compound.
Correct Option: (2)

Question 12: Which of the following is a copolymer?
(1)


(2)



(3)






(4)

Chapter: Polymers
Level of Difficulty: Easy
Solution
Homopolymers are formed by polymerization of the same type of monomers. Copolymers are formed
when a mixture of more than one monomeric species is allowed to polymerize. For example, Buna-N is a
copolymer of acrylonitrile and 1,3-butadiene. From the given options, option (2) represents a
copolymer.
Correct Option: (2)

Question 13: Which of the following carbocations will not rearrange?
(1) (2) (3) (4)
Chapter: Some Basic Principles in Organic Chemistry
Level of Difficulty: Moderate
Solution
The rearrangement of a carbocation is possible only if it can form more stable carbocation. Sometimes,
it can also release angle strain present in relatively smaller or bigger rings. Hence, (1), (2) and (4)
rearrange as shown below. The compound in (3) cannot rearrange because more stable carbocation is
not possible by shifting alkyl or halide group.
(1) (2)


(4)
Correct Option: (3)

Question 14: The correct order of stability of the following free radicals is


(1) III > I > II > IV (2) I > II > IV > III (3) IV > II > I > III (4) I > III > II > IV
Chapter: Some Basic Principles in Organic Chemistry
Level of Difficulty: Easy
Solution
The free radicals are electron-deficient species, whose stability is enhanced by electron donating groups.
Therefore in (III), allylic free radical is most stable since electron density can be transferred by both
inductive as well as resonance effects. As (I), (II) and (IV) are 3, 2 and 1 free radicals, respectively, the
correct order of stability is III > I > II > IV.
Correct Option: (1)

Question 15: Which of the following sets of quantum numbers is not representing the electrons which
are eliminated from Fe to convert it into Fe
3+
?
(1) n = 4, l = 0, m = 0, s = 1/2 (2) n = 4, l = 0, m = 0, s = 1/2
(3) n = 3, l = 2, m = 0, s = 1/2 (4) n = 3, l = 1, m = 0, s = 1/2
Chapter: Atomic Structure
Level of Difficulty: Moderate
Solution
Ground state electronic configuration of Fe: 1s
2
2s
2
2p
6
3s
2
3p
6
3d
6
4s
2

The electronic configuration of Fe
3+
: 1s
2
2s
2
2p
6
3s
2
3p
6
3d
5
4s
0

Hence, two electrons are removed from the fourth shell and the third shell. Two types of orbitals (d and
s) are involved. The value of l for s-orbital is 0 and for d-orbital, l = 2.


Fourth set of quantum numbers contains l = 1, that is, p-orbital. The p-orbitals are not involved in
ionization.
Correct Option: (4)
Quick Tip: Just look for the electrons removed from the atom, and from which type orbitals are involved
in this process.

Question 16: For which of the following equilibrium, the value of K
p
is not greater than the value of K
C
?
(1)
2
2C(s) O (g) 2CO(g) + (2)
2 4 2
2C(s) 2H O(g) CH (g) CO (g) + +
(3)
4 2 4 2
CuSO 5H O(s) CuSO (s) 5H O(g) + (4)
2 2
Mg(OH) (s) MgO(s) H O(g) +
Chapter: Chemical Equilibrium
Level of Difficulty: Easy
Solution
( )
n
p C
K K RT
A
= ; where An = product side gaseous moles reactant side gaseous moles.
(1) 2 1 1 ( )
p C
n K K RT A = = = (2)
0
2 2 0 ( )
p C p C
n K K RT K K A = = = =
(3)
5
5 0 5 ( )
p C
n K K RT A = = = (4) 1 0 1 ( )
p C
n K K RT A = = =
Correct Option: (2)

Question 17: The correct order of increasing covalent character of the following is
(1) KCl < CaCl
2
< AlCl
3
< SiCl
4
(2) SiCl
4
< AlCl
3
< CaCl
2

< KCl
(3) AlCl
3
< CaCl
2

< KCl < SiCl
4
(4) CaCl
2

< SiCl
4
< KCl < AlCl
3

Chapter: Chemical Bonding and Molecular Structure
Level of Difficulty: Easy
Solution
According to Fajans rule, as the charge on the cation increases, its effective nuclear charge as well as
polarizing power increases. Hence, covalent character will also increase. The increasing order should be
KCl < CaCl
2
< AlCl
3
< SiCl
4
.
Correct Option: (1)

Question 18: The correct order of increasing CO bond length of
2
3 2
CO, CO , CO

is
(1)
2
2 3
CO CO CO

< < (2)
2
3 2
CO CO CO

< <


(3)
2
2 3
CO CO CO

< < (4)


2
3 2
CO CO CO

< <

Chapter: Chemical Bonding and Molecular Structure
Level of Difficulty: Moderate
Solution
All bonds in
2
3
CO

are equivalent because it is resonance hybrid.

So, the bond order of
2
3
CO

is 4/3.
Bond order in CO is 3 and of CO
2
it is 2.
Since bond length is inversely proportional to the bond order, bond length order is
2
2 3
CO CO CO

< <
Correct Option: (1)

Question 19: The reaction of tin metal with acid can be written as
2+
2 o
2
Sn /Sn
Sn(s) 2H (aq) Sn (aq) H (g); 0.14 V E
+ +
+ + =
Assume that
2+
[Sn ] 1M = and the partial pressure of hydrogen gas is 1 atm, then the
(1) cell reaction is spontaneous at pH = 5.
(2) cell reaction is non-spontaneous at standard conditions.
(3) cell reaction is spontaneous at pH = 2.
(4) cell reaction is spontaneous for all pH values.
Chapter: Electrochemistry
Level of Difficulty: Hard
Solution
2
2
Sn(s) 2H (aq) Sn (aq) H (g)
+ +
+ +
Nernst equation for this cell reaction:
2
2
H o
cell cell 2
[Sn ]
ln
[H ]
p
RT
E E
nF
+
+

=
At standard temperature;
2
2
H o
cell cell 2
[Sn ]
0.0591
ln
[H ]
p
E E
n
+
+

=
+ 2+
2
o o o
cell
H /H Sn /Sn
0 ( 0.14 V) 0.14 V E E E = = =
C
O
O
O
C
O
O
O
C
O
O
O


For pH = 5,
cell 5 2
0.059 1
0.14 log 0.16 V
2 (10 )
E

= =
For standard state;
o
cell cell
0.14 V E E = =
For pH = 2,
cell 2 2
0.059 1
0.14 log 0.14 0.12 0.02 V
2 (10 )
E

= = =
Correct Option: (3)

Question 20: Select the correct statement about compressibility factor of one mole gas which is kept
inside a 22.4 L vessel at 273 K.
(1) For Z > 1 the pressure of gas will be less than 1 atm.
(2) For Z > 1 the pressure of gas will be more than 1 atm.
(3) If b dominates, pressure will be less than 1 atm.
(4) If a dominates, pressure will be greater than 1 atm.
Chapter: Gaseous State
Level of Difficulty: Hard
Solution
Using ideal gas law:
1 0.082 273
1atm
22.4
nRT
p
V

= = =
So if the gas is ideal, then its pressure is 1 atm. However, it is a real gas, so at different conditions its
pressure may vary from the ideal pressure.
If its compressibility factor Z > 1, this indicates that repulsive forces dominate inside the gas. These
repulsive forces imply that the pressure should be more than ideal pressure, that is, more than 1 atm
pressure.
If b dominates, then
m
m
( ) 1
pV pb
p V b RT Z
RT RT
= = = +
This equation implies that Z will be more than one always, and pressure should be more than 1 atm.
If a dominates, then
m
m 2
m m
1
pV a a
p V RT Z
V RT RTV
| |
+ = = =
|
\ .

This equation implies that Z < 1, so the pressure should be less than 1 atm as the attraction forces will
dominate here.
Correct Option: (2)



Question 21: The precipitate of CaF
2
(K
sp
= 1.7 10
10
) is observed when equal volumes of the following
are mixed.
(1)
4 2+ 4
10 M Ca + 10 MF (2)
2 2+ 3
10 M Ca + 10 MF
(3)
5 2+ 3
10 M Ca + 10 MF (4)
3 2+ 5
10 M Ca + 10 MF
Chapter: Ionic Equilibrium
Level of Difficulty: Moderate
Solution
The reaction is
2
2
CaF Ca 2F
+
+
The solubility product is
2 2 10
sp
[Ca ][F ] 1.7 10 K
+
= =
The ionic product is
2 2
I
[Ca ][F ] K
+
=
(1)
2 2 4 4 2 13
I
[Ca ][F ] 0.5 10 (0.5 10 ) 1.25 10 K
+
= = =
(2)
2 2 2 3 2 9
I
[Ca ][F ] 0.5 10 (0.5 10 ) 1.25 10 K
+
= = =
(3)
2 2 5 3 2 12
I
[Ca ][F ] 0.5 10 (0.5 10 ) 1.25 10 K
+
= = =
(4)
2 2 3 5 2 14
I
[Ca ][F ] 0.5 10 (0.5 10 ) 1.25 10 K
+
= = =
In the case of (2), ionic product is greater than solubility product, so precipitation will be observed in this
case.
Correct Option: (2)

Question 22: Consider the decomposition of gaseous N
2
O
5
;
2 5 2 2
2N O 4NO (g) O (g) + . At moderate
gas pressures, it follows the mechanism given below:
1
1
2 5 2 3
N O NO +NO
k
k


2
2 3 2 2
NO NO NO NO O
k
+ + +
3
3 2
NO NO 2NO
k
+
Which of the following statements is correct?
(1) Overall order of the reaction is 2.0 (2) Order w.r.t. N
2
O
5
is 1.0
(3) Order w.r.t. NO
3
is 1.0 (4) Order cannot be predicted
Chapter: Chemical Kinetics


Level of Difficulty: Hard
Solution
The molecule NO
3
is an intermediate whose concentration is small and can be calculated by the steady
state approximation. Same can be said for NO.
Rate of production of NO = Rate of destruction of NO
2
2 2 3 3 3 2
3
[NO ][NO ] [NO][NO ] [NO] [NO ]
k
k k
k
= =
For NO
3
, rate of production of NO
3
= rate of destruction of NO
3

1 2 5 1 2 2 2 3 3
[N O ] ( [NO ] [NO ] [NO])[NO ] k k k k

= + +
1 2 5
3
1 2 2 2
[N O ]
[NO ]
[NO ] 2 [NO ]
k
k k

=
+

for the steady state condition of NO
3
.The rate of production of oxygen, which is also the rate of reaction,
is
2
2 2 3
d[O ]
[NO ][NO ]
d
k
t
=
Substituting the value of [NO
3
], we get
1 2 2 5 2
2 5
1 2
[N O ] d[O ]
[N O ]
d 2
k k
k
t k k

' = =
+

where
1 2
1 2
2
k k
k
k k

' =
+
.
Correct Option: (2)

Question 23: Which of the following statements is correct about the defects in the crystals?
(1) Schottky defects are observed in the compounds in which there is a large difference in the size of
cation and anion, whereas Frenkel is observed when cation and anion are approximately of the same
size.
(2) Schottky defects disturb the ratio of cations and anions, whereas Frenkel defects maintain the ratio.
(3) Schottky defects lead to lowering in density, whereas Frenkel defects do not affect the density.
(4) Both interstitial defects and Frenkel defects cause increase in the density of solid.
Chapter: Solid State
Level of Difficulty: Moderate
Solution


Schottky defect is observed in ionic solids in which cationic and anionic sizes are approximately the
same. It arises when equal number of cations and anions are missing from their position. It causes
decrease in the density of solid.
Frenkel defect is observed in ionic solids in which cationic and anionic size difference is large. It arises
when the smaller ion leaves its original place and moves into interstitial spaces. This causes no change in
cationanion ratio, and hence no change in the density.
Interstitial defect arises when a constituting particle is also placed into interstitial space. This causes an
increase in density.
Correct Option: (3)

Question 24: The solubility of pure oxygen in water at 20C and 1 atm pressure is 43 mg O
2
per liter of
water. When air is in contact with water and the air pressure is 585 torr at 20C, how much amount of
oxygen is dissolved in 500 ml water? The average concentration of oxygen in air is 21.1%(V/V).
(1) 7 mg (2) 3.5 mg (3) 16.5 mg (4) 33 mg
Chapter: Solutions
Level of Difficulty: Moderate
Solution
From Henrys law:
1 1
gas H gas
2 2
C p
C K p
C p
= =
Given that C
1
= 43 mg/l; p
1
= 1 atm = 760 torr; C
2
= ?; p
2
=
21.1
585 123.435 torr
100
=
Substituting in the above expression, we get
2
123.44 43
mg/L 7 mg/L
760
C

= =
Therefore,
2
7
Mass of O in 500 ml water 500 3.5 mg
1000
= =
Correct Option: (2)

Question 25: A saturated solution is prepared by dissolving 0.2 g of polypeptide in water to give 500 mL
of solution. The solution has an osmotic pressure of 3.74 torr at 27C. What is the approximate
molecular mass of the polypeptide?
(1) 8559 (2) 2000 (3) 180 (4) 203
Chapter: Solutions


Level of Difficulty: Easy
Solution
The osmotic pressure is given by
/ w M
CRT RT
V
H = =
Therefore,
0.2 0.082 300
2000
3.74
0.5
760
wRT
M
M

= = ~
H


Correct Option: (2)

Question 26: When a solution of acetic acid and water was shaken with charcoal, part of acetic acid was
absorbed by charcoal and its concentration decreased. The relation between the adsorption of acetic
acid from the solution and initial concentration of the acetic acid is obtained by plotting log(x/m) vs. log
C, which gives a straight line with slope:
(1) n (2) 1/n (3) k (4) log k
Chapter: Surface Chemistry
Level of Difficulty: Easy
Solution
According to the Freundlich isotherm for adsorption in liquid solutions,
1/ n
x
kC
m
=
Taking log of this equation,
1
log log log
x
k C
m n
| |
= +
|
\ .

Now comparing this equation with straight line equation, y mx c = + , it tells us that the graph between
log and log
x
C
m
| |
|
\ .
produces straight line with slope equal to 1/n and intercept on y-axis equal to log(k).
Correct Option: (2)

Question 27: The reaction
o
2 2 2
2N O(g) 2N (g) O (g); 163kJ H + A = . What is the change in internal
energy for the decomposition of 176 g of N
2
O at 25C?
(1) 662 kJ (2) 657 kJ (3) 642 kJ (4) 168 kJ
Chapter: Chemical Thermodynamics
Level of Difficulty: Easy
Solution


We have
o o o
g g
H U n RT U H n RT A = A + A A = A A
Moles of N
2
O taken =
176
4 mol
44
=
Therefore,
o
Moles Change for one mole 4 ( )
g
U H n RT A = = A A
or
8.314 300
4 163 1 kJ 662 kJ
1000
U
| |
A = ~
|
\ .

Correct Option: (1)

Question 28: Which of the following statements is not correct?
(1) Final temperature in reversible adiabatic expansion is greater than that in irreversible adiabatic
expansion.
(2) When heat is supplied to an ideal gas in isothermal process, kinetic energy of gas remains constant.
(3) When an ideal gas is subjected to adiabatic expansion, it gets cooled.
(4) Entropy increases when an ideal gas expands isothermally.
Chapter: Chemical Thermodynamics
Level of Difficulty: Hard
Solution
For adiabatic process,
V
U w w C T A = = A
V
w
T
C
A =
Since work done in reversible process is more than that in irreversible process, the temperature change
is more in reversible process, that is, the temperature decrease will be more.
Since kinetic energy of ideal gases is proportional to root of temperature, for isothermal it will not
change.
From the above relations, it is clear that when an ideal gas is subjected to adiabatic expansion, its
temperature decreases.
Entropy increases when an ideal gas expands isothermally because increasing the volume causes
increase in randomness.
Correct Option: (1)

Question 29: Ammonium nitrate detonates above 300C according to the chemical equation given
below:
4 3 2 2 2
NH NO N O H O + +


Select the correct statement:
(1) The volume change is approximately 3.5 times the initial volume at STP.
(2) The volume change is approximately 1.5 times the initial volume taken at 323C.
(3) On explosion of 80 g of ammonium nitrate, 78.4 L of total volume is obtained at STP.
(4) 100 g of NH
4
NO
3
produces volume of 215.3 L of the total gases at 323C and 1 atm pressure.
Chapter: Some Basic Concepts of Chemistry
Level of Difficulty: Moderate
Solution
The balanced form of equation is:
4 3 2 2 2
2NH NO (s) 2N (g) O (g) 4H O(g) + +
Since the reactant is in solid state, so the volume occupied by it is negligible.
The moles are approximately 3.5 times, but the volume is not 3.5 times. At STP, water is liquefied so
volume is reduced and corresponds to the gaseous moles left.
Taking 80 g of NH
4
NO
3
(one mole) will produces 3.5 mol
But at STP, gaseous moles are 1.5 mol, so volume produced is approximately 1.5 22.4 L 33.6 L =
100 g of NH
4
NO
3
contains
100
1.25 mol
80
=
Therefore, gaseous moles appeared at given condition 3.5 1.25 4.375 mol = =
The volume of gases,
3.5 1.25 0.082 600
215.3 L
1
nRT
V
p

= = ~
Correct Option: (4)

Question 30: The presence of halogen, in an organic compound, is detected by
(1) iodoform test. (2) Tollens reagent. (3) Beilsteins test. (4) Millons test.
Chapter: Purification and Characterization of Organic Compounds
Level of Difficulty: Easy
Solution
In Beilstein test, a (clean) copper wire is heated in the non-luminous flame of the Bunsen burner until it
ceases to impart any green or bluish green color to the flame. The heated end of the wire is dipped into
the organic compound and is again introduced into the Bunsen flame. The appearance of a bluish green
or green flame due to the formation of volatile cupric halides shows the presence of halogens in the
given organic compound.
Correct Option: (3).


Quick Tip: Compounds like urea, thiourea that do not contain halogen give positive Beilsteins test.
Fluoride ion is not detected by this test as copper fluoride is not volatile. Hence, it is not a very
dependable test, and thus, negative Beilstein test is more useful than a positive one.

Potrebbero piacerti anche